• Janus
    15.6k
    If I may interject a question as someone with only a superficial understanding of Kant...

    Isn't it a bit of an overstatement to say we know *nothing* of the thing-in-itself? Why not a more nuanced view, in which we know a limited amount about things-in-themselves, but some of us know more than others, depending on the thing under consideration.
    wonderer1

    The thing in itself is posited as being what gives rise to the thing as it appears. we cannot be conscious of the primary effect on us of things, we can only be conscious of how we come to represent those things to ourselves.

    Imagine the universe without humans; what would it be like? Of course, we have no idea, except to say that it wouldn't be like anything, but if we were there it would appear in just the kinds of ways it does, or so we imagine.

    Look at it another way: is the world in itself a concatenation of energetic structures that are somehow isomorphic to the structures we perceive? Or is it a world of things that look just like the things as they appear to us. The former seems like a reasonable conjecture, but the latter is naive realism and in view of our scientific understanding of perception and even just on critical analysis seems absurd. If the latter were to be the case, then it would seem there must be a God or universal mind to whom things (which are ideas in its mind) look to it, just as they look to us. Which would make our senses windows onto the mind of God, or some such thing.
  • Mww
    4.6k
    the true origin of our proofs in pure math is a priori in the sense of our faculty of reasonBob Ross

    The true origin of the possibility of our proofs, is in reason and is a priori.
    The origin of the proofs themselves, is in understanding, and is a posteriori.

    our proofs (…) of the useful application of math is a priori in the sense of our faculty of reason’s ability to construct the phenomenal world according to principles.Bob Ross

    Useful application…..is empirical, for which the phenomenal is constructed, but by understanding, according to conceptions. Understanding is incompetent to construct synthetic principles a priori, but only to construct the conceptions and the synthesis of them to each other, representing the content of those principles. Transcendental application, is neither useful nor empirical, the form of which is merely syllogistic and thus having no empirical content.

    “…. The understanding may be a faculty for the production of unity of phenomena by virtue of rules; reason is a faculty for the production of unity of rules (of the understanding) under principles. Reason, therefore, never applies directly to experience, or to any sensuous object; its object is, on the contrary, the understanding, to the manifold cognition of which it gives a unity à priori by means of conceptions, and which is of a nature very different from that of the unity produced by the understanding….”
    —————

    And because logic is a metaphysical practice, and the conception is already a methodological requirement, then it could be said that they are metaphysically necessary.
    -Mww

    Metaphysical necessity is essentially that it is true in all possible worlds
    Bob Ross

    Jeeezz, I hate that expression. Like…..what other world is there? That other worlds are not impossible says not a gawddamn thing about the one we’re in. And we’re not in a possible world; we’re in a necessary world.

    Metaphysically necessary merely indicates a condition in a thinking subject. End of story.
    —————

    A cautionary tale, relevant to the thread topic:

    “…. The success which attends the efforts of reason in the sphere of mathematics naturally fosters the expectation that the same good fortune will be its lot, if it applies the mathematical method in other regions of mental endeavour besides that of quantities. Its success is thus great, because it can support all its conceptions by à priori intuitions and, in this way, make itself a master, as it were, over nature; while pure philosophy, with its à priori discursive conceptions, bungles about in the world of nature, and cannot accredit or show any à priori evidence of the reality of these conceptions.

    As we have taken upon us the task of determining, clearly and certainly, the limits of pure reason in the sphere of transcendentalism, and as the efforts of reason in this direction are persisted in, even after the plainest and most expressive warnings, hope still beckoning us past the limits of experience into the splendours of the intellectual world—it becomes necessary to cut away the last anchor of this fallacious and fantastic hope. We shall, accordingly, show that the mathematical method is unattended in the sphere of philosophy by the least advantage—except, perhaps, that it more plainly exhibits its own inadequacy—that geometry and philosophy are two quite different things, although they go hand in hand in the field of natural science, and, consequently, that the procedure of the one can never be imitated by the other.…”

    This just says, while mathematics is that which exhibits absolute certainty, and we are ourselves the author of mathematical procedures, then it is true absolute certainty is possible for us. The cautions lay in thinking that insofar as absolute certainty is possible, we are thus authorized to pursue the experience of some object representing it. But that just won’t work, because the objects being pursued are not those we construct of ourselves, but are thought to exist in their own right. And they might, but there are no mathematically derived principles given from pure reason, and by association there can be no absolute certainty contained therein, that can support the reality of that object.

    The certainty of mathematics can not be imitated in philosophy.

    Question: Is a universal mind an absolute certainty deduced from mathematical principles? If not, the object, represented as a universal mind in our understanding, is a mere philosophical possibility. If all our representations are derived from ideas contained in that which is not itself a certainty, why should we trust that our representations arise from it?

    I dunno, man. If I can grasp that all my representations belong to me, and never doubt or question that they do, why would I shadow that certainty with that which has decidedly less so, by thinking to myself that my representations are merely offshoots of something else?

    While you are correct in saying it is possible, what’s missing is why I should even consider the possibility that analytic idealism holds more persuasions than the transcendental idealism I currently endorse?

    So…..what do I gain by granting my representations have their irreducible origin somewhere other than in me?
  • Bob Ross
    1.2k


    Hello Janus,

    Fer fuck's sake, Bob, how many times do I have to tell you I'm not claiming that object permanence or independence is a feature of, or inference about, anything more than the phenomenal world of human experience.

    Firstly, Janus, I don’t know why you are getting so hostile. I am trying to have a good faith conversation with you about your perspective on Kantianism, and all you are doing is insulting me left and right. Relax my friend! If there is something I am misunderstanding about your view or misrepresenting, then please always feel free to point it out and I will try to re-understand what you are saying. There’s no need to insult one another (:

    So, I didn’t understand you to be claiming that the ‘object permanence’ only pertains to the phenomenal world for you, in terms of what you would claim to know. Originally I do not feel that you claimed that:

    Object permanence is inferred on account of the experienced invariance of objects. It is an inductive, that is fallible, inference, not a deductive, infallible inference.

    As you said the above and ‘object permanence’ typically refers to the claim that the objects persist in the context beyond human representations of the world (which would be beyond the phenomenal one).

    Anyways, it doesn’t matter, because you are now clarifying that by it you mean something different—something like us using object permanence as it pertains only to the phenomenal world.

    Here’s my problems prima facie with this claim:

    1. The idea that object persist in the phenomenal world doesn’t make sense unless you are claiming that the phenomenal world are not representations but, rather, productions of your mind or a thing-in-itself. The Kantian idea of the phemonenal world is that just that of the representations we have—object permanence in those representations would entail that there is something which you perceive, something you represent, which continues to exist wherever you last perceived it which would entail it is outside of your mind and thusly in the things-in-themselves.

    2. If you say that your phenomena suggest that there is object permanence, and #1 is true, then you are equally conceding that you can know of the things-in-themselves beyond merely that they exist (even if it is inductive or abductive reasoning).

    You don't pay attention to anything I write, apparently, or else you distort it in the reading. I've already explained numerous times that everything I have been saying relates only to the phenomenal world. When is that going to sink in?

    I understand that you are claiming you can only know of the phenomenal world, and I am trying to show you that it is a self-undermining argument (in terms of Kant’s argumentation at least). For example, see #1 above.

    It’s not that I am ignoring what you are saying, I am challenging it.

    Our representations of the phenomenal world are neither completely accurate nor completely inaccurate; a fact which has no bearing whatsoever on the question of whether we know the world as it is in itself (which simply as a matter of definition we don't, because anything we know is by definition the world as it is for us).

    Again, here’s another similar problem to #1 above (I think) with your view: you can’t claim any bearings on accuracy or inaccuracy if you can’t know anything about the things-in-themselves; and if you want to use ‘accuracy’ and ‘inaccuracy’ to refer to the phenomenal world, then you are claiming only that some phenomena are more useful then other phenomena. The moment you claim that a set of phenomena are truly accurate or inaccurate to any degree you are thereby comparing them to something other than phenomena, and that is, by definition, the things-in-themselves. I think to be a Kantian there are much more commitments to be consistent than Kant wanted.

    No Bob, those minds may be a part of the world in itself, but the mind as we know it is the mind as it appears to us. Kant's twelve categories are analytically determined by reflecting on the ways in which we understand phenomenal objects.

    I am not talking about the brain. I am talking about those categories Kant’s comes up with that are a part of your representative faculty. Unless you want to claim that your representative faculty exist as nothing then you will have to concede that they exist as a thing-in-itself.

    More unargued assertion; it's not interesting, Bob

    It is all metaphysics because metaphysics is the study of that which is beyond the possibility of all experience—and transcendental philosophy is all about trying to infer the representative faculties from experience but those faculties are necessarily beyond all possible experience as the necessary preconditions thereof.

    Kant does not argue for a soul, at least not in the CPR.

    I think, given his background, he was; but, at the very least, he argued for a thing-in-itself ‘I’ of the synthetic unity of apperception. See ‘Of the Originally Synthetical Unity of Apperception’ section 12 of CPR for the whole argument, but here’s an excerpt:

    The “I think” must accompany all my representations, for otherwise something would be represented in me, which could not be thought; in other words, the representation would either be impossible, or at least be, in relation to me, nothing...All the diversity or manifold content of intuition has, therefore, a necessary relation to the “I think,” in the subject in which this diversity is found.

    However, if the “I think” exists, it is a thing-in-itself—not phenomena. And if it is a thing-in-itself, then Kant is contradicting his claim that we cannot know about the things-in-themselves.

    Bob
  • Bob Ross
    1.2k


    Hello Mww,

    The true origin of the possibility of our proofs, is in reason and is a priori.
    The origin of the proofs themselves, is in understanding, and is a posteriori.

    I thought the origin of the proofs themselves, being in the understanding, would be a priori, would they not?

    Or are you claiming that the possibility for math is within our understanding (and thusly a priori), but that we cannot know a priori the mathematical relations of objects a priori?

    If so, then I agree.

    Also, just a side note, I am a mathematical anti-realist; so, for me, math is not a priori in the sense of being a part of our construction, via the understanding, of the world around us. But I understand from a Kantian view what you are saying.

    Useful application…..is empirical, for which the phenomenal is constructed, but by understanding, according to conceptions. Understanding is incompetent to construct synthetic principles a priori, but only to construct the conceptions and the synthesis of them to each other, representing the content of those principles. Transcendental application, is neither useful nor empirical, the form of which is merely syllogistic and thus having no empirical content.

    Agreed.

    Jeeezz, I hate that expression. Like…..what other world is there? That other worlds are not impossible says not a gawddamn thing about the one we’re in. And we’re not in a possible world; we’re in a necessary world.

    It isn’t that the possible worlds exist but, rather, that under one’s metaphysical commitments there is an existence with the potency to actualize the thing, and as such the thing is considered metaphysically possible.

    We can be more certain of logical possibility than metaphysical possibility, and actual possibility less than the former and more than the latter.

    Metaphysically necessary merely indicates a condition in a thinking subject. End of story.

    If by ‘condition’ you mean the belief that there is an existent thing which has the potency to actualize the said thing, then I agree.

    This just says, while mathematics is that which exhibits absolute certainty, and we are ourselves the author of mathematical procedures, then it is true absolute certainty is possible for us.

    I don’t see how this is true if the application of math is a posteriori. There is no absolute certainty in that, unless you are claiming that our constructions of the same things-in-themselves will never waver.

    The cautions lay in thinking that insofar as absolute certainty is possible, we are thus authorized to pursue the experience of some object representing it. But that just won’t work, because the objects being pursued are not those we construct of ourselves, but are thought to exist in their own right. And they might, but there are no mathematically derived principles given from pure reason, and by association there can be no absolute certainty contained therein, that can support the reality of that object.

    I feel like you are agreeing with me on my previous statement here: correct?

    The certainty of mathematics can not be imitated in philosophy.

    When you say we have absolute certainty in math, are you just referring to what we can know via our faculty of reason? Like, theoretically, we can be absolutely certain that 1 + 1 = 2 without appeal to the empirical world?

    Is a universal mind an absolute certainty deduced from mathematical principles?

    I don’t think we can deduce any existent thing from math. Math just doesn’t afford that information, nor does logic.

    If not, the object, represented as a universal mind in our understanding, is a mere philosophical possibility

    The universal mind isn’t represented to us as a thing-in-itself--it is the thing-in-itself and, as such, is the substrate of the reality in which we live which is what we represent to ourselves: not the Universal Mind.

    If all our representations are derived from ideas contained in that which is not itself a certainty, why should we trust that our representations arise from it?

    We can’t be certain of virtually anything. We can’t be certain that there is a law of gravity, that math applies to objects in so and so manner, that logic applies to objects in so and so manner, etc. I am a neo-schopenhauerian and, as such, am a neo-neo-Kantian.

    I have no problem admitting that we cannot be certain in metaphysics whatsoever—including Transcendental Philosophy. I, nor you, can be certain that there are twelve categories of the understanding, or that the understanding constructs the representative world around us. So I just don’t see why this is a problem I guess.

    If I can grasp that all my representations belong to me

    But I would say we can get at, just like how metaphysically your representations are of your mind, other aspects metaphysically of reality. Just because you are representing things doesn’t mean that we are barred from investigating the things-in-themselves.

    And, also, I feel like in order for your argument here to work, that you have to concede that your mind is a thing-in-itself. Otherwise, I have no clue what you mean when you say that it belongs to you.

    and never doubt or question that they do

    There’s no certainty in the claim that your conscious experience is a representation produced by your mind: it is based off of a cumulative evidential argument. It explains the data of your experience the best. If you have some proof that provides absolute certainty, then please share it!

    why would I shadow that certainty with that which has decidedly less so, by thinking to myself that my representations are merely offshoots of something else?

    Because, just like how it makes the most sense that your mind is representing the real world, it makes sense to account for object permanence and other minds as within an objective world and that that objective world is fundamentally mind.

    While you are correct in saying it is possible, what’s missing is why I should even consider the possibility that analytic idealism holds more persuasions than the transcendental idealism I currently endorse?

    Because, in my opinion, the transcendental notion that we cannot know the things-in-themselves is self-undermining: your mind is a thing-in-itself and transcendental philosophy is an evidence based analysis thereof. It is doing nothing special in comparison to the rest of good metaphysics. And this is barring you, I think, from fleshing out your whole metaphysical view (because you think that you simply cannot know the things-in-themselves). What’s missing is your account of the rest of the world.

    For example, under transcendental idealism I don’t think you can claim:

    1. There are other minds.
    2. That you have a mind.
    3. That you have representative faculties.
    4. That objects persist in their existences even when you are not perceiving them.
    5. Etc…

    So…..what do I gain by granting my representations have their irreducible origin somewhere other than in me?

    I think a more plausible explanation and account of reality. I can account for object permanence, other minds, my mind, my representative faculties, etc. no problem: I don’t think you can.

    Bob
  • Janus
    15.6k
    Firstly, Janus, I don’t know why you are getting so hostile.Bob Ross

    I'm not becoming hostile, just impatient. I just don't believe that you are grasping what is meant by things in themselves. So, I am not going to deal with or respond to anything other than that one point at this juncture.

    As you said the above and ‘object permanence’ typically refers to the claim that the objects persist in the context beyond human representations of the world (which would be beyond the phenomenal one).Bob Ross

    The idea of things-in-themselves is not meant to be interpreted as claiming that there are things just like those that are perceived that exist independently of human perception; the "thing" in there is a kind of placeholder for some unknowable X. So, object permanence cannot reasonably be thought to apply to things in themselves, except in the sense that whatever it is that appears to us as invariant objects does so reliably, which suggests, but doesn't prove, that the in itself is invariant.

    In truth, we never perceive whole objects, but only views of them from different perspectives, so we construct the notion of whole objects from the various views (and feels) we have of them, and the fact that we can act on them, and the whole picture of a world of objects of more or less invariance is woven together with remarkable consistency by the brain/ mind. Part of this picture consists in the idea of object invariance; this idea is inevitable, even animal behavior shows that they expect objects not to simply disappear when they can't be seen. I observe this when I throw the ball for my dog and it inadvertently goes into the long grass; he never stops searching for it until he finds it showing that he expects it to be there somewhere and not to have simply disappeared.

    So, I haven't been arguing that it is provable that the in itself is invariant or that phenomenal objects are "permanent", but that object permanence is the inference to the best explanation in the empirical context, and that regarding noumenal invariance we really have no idea how to assess which explanation would be the more plausible because we have nothing to compare any explanation with. So, if anyone says that they think this or that metaphysical explanation is the most plausible, that really only speaks to their own personal preferences. That, in short, is all I've been arguing for.
  • Srap Tasmaner
    4.6k
    In truth, we never perceive whole objects, but only views of them from different perspectives, so we construct the notion of whole objects from the various views (and feels) we have of themJanus

    But -- to start with, wholes and views aren't opposites; they're different sorts of things altogether, and it's exactly this ambiguity that's troublesome.

    So would you rather say we perceive partial objects, out of which we construct a whole in our minds, conceptually, or that we have views of (presumably whole and complete) objects? I've substituted "have" there, but you can stick to "perceive views" if you intended to treat a view as a sort of object.

    If the model is that there's something out there, and then our sensorium, and then finally, at the greatest remove from what's out there, our intellect, are phenomena the input to the sensorium, or the output of the sensorium? I'm thinking it's output, which is to say, the input for the intellect.

    But that too is ambiguous, and if we expect this account to align with the findings of neuroscience, we have to decide whether to count the processing of perceptual data as part of the intellect or part of the sensorium. If you say intellect, then phenomena are almost nothing, the firing of neurons without considering where those impulses go (must go). But if you say sensorium, then an awful lot has already been done, without your awareness, before it reaches the intellect.

    And that's fine, still seems like this is the way to go because that signals processing isn't incidentally unconscious but necessarily so, and we get to call phenomena whatever the first things are that we even can become aware of, whether we happen to be or not.

    But at what point do we get objects? That's the question. Does perception make available to awareness uninterpreted views? That looks unlikely. Color constancy suggests that whether something is an object determines how its color is presented to your awareness, and you have no control over this. It seems your perceptual apparatus is already making decisions about which parts of your so-called field of vision are objects, or anyway something has.

    And if objects are only offered to your awareness pre-assembled, we might say, then objects are constitutive of phenomena, not the other way around. The alternative is to take intellect to include this unconscious processing, but then I'm really not clear what phenomena are supposed to be. Not views certainly. Not color patches. I really don't know what.
  • Janus
    15.6k
    But -- to start with, wholes and views aren't opposites; they're different sorts of things altogether, and it's exactly this ambiguity that's troublesome.Srap Tasmaner

    Imagine you have an extremely limited view of something, so limited that you cannot tell what it is you are looking at. Say you are looking through binoculars and you see a patch of brown, then you change the focal length, and you realize it is a brick, then you change it gain and you realize it is a wall of bricks and changing it again you see that the wall is a wall of a building. Are you seeing a building now? Perhaps not, as it turns out it is just a freestanding facade.

    So would you rather say we perceive partial objects, out of which we construct a whole in our minds, conceptually, or that we have views of (presumably whole and complete) objects? I've substituted "have" there, but you can stick to "perceive views" if you intended to treat a view as a sort of object.Srap Tasmaner

    It seems to me we could say that any sensory perception of anything that we recognize as being anything at all would count as a perception of a particular, and particular kind, of object iff it turns out that our recognition is not mistaken.

    If the model is that there's something out there, and then our sensorium, and then finally, at the greatest remove from what's out there, our intellect, are phenomena the input to the sensorium, or the output of the sensorium? I'm thinking it's output, which is to say, the input for the intellect.Srap Tasmaner

    Since we are not conscious of the process of input to the sensorium, I would agree that phenomena (defined as recognizable sense objects) are the output, so it seems we agree on that.

    But that too is ambiguous, and if we expect this account to align with the findings of neuroscience, we have to decide whether to count the processing of perceptual data as part of the intellect or part of the sensorium. If you say intellect, then phenomena are almost nothing, the firing of neurons without considering where those impulses go (must go). But if you say sensorium, then an awful lot has already been done, without your awareness, before it reaches the intellect.Srap Tasmaner

    I don't see a separable intellect in the process. The idea of the intellect is the idea of a "faculty", an idea which probably does not map well onto a neuroscientific understanding of the brain. It seems to be a kind of reification. Taking vision as paradigmatic, the instant reflected light enters the eye, the proceeds has begun, and it (in the right circumstances) terminates with recognition. But we are conscious only of the recognition (or lack of it). And reading on I see you seem to be saying something similar here:

    And that's fine, still seems like this is the way to go because that signals processing isn't incidentally unconscious but necessarily so, and we get to call phenomena whatever the first things are that we even can become aware of, whether we happen to be or not.Srap Tasmaner

    But at what point do we get objects? That's the question. Does perception make available to awareness uninterpreted views? That looks unlikely. Color constancy suggests that whether something is an object determines how its color is presented to your awareness, and you have no control over this. It seems your perceptual apparatus is already making decisions about which parts of your so-called field of vision are objects, or anyway something has.Srap Tasmaner

    I think perception sometimes presents us with uninterpreted views, but generally not uninterpreted views of familiar objects (unless it is a very unusual circumstance like we have taken a psychedelic for example). If psychedelics switch off the part of the brain that recognizes familiar objects, allowing other parts of the brain which normally do not communicate with one another to communicate, then such an experience of being unable to tell what you were looking at would seem to make sense.

    And if objects are only offered to your awareness pre-assembled, we might say, then objects are constitutive of phenomena, not the other way around. The alternative is to take intellect to include this unconscious processing, but then I'm really not clear what phenomena are supposed to be. Not views certainly. Not color patches. I really don't know what.Srap Tasmaner

    Yes, but as my looking-through-the-binoculars example shows, there is always an ambiguity as to just what is the object; is it a brown patch, a brick, a wall, a building or a free-standing facade? That information may not be given just in the view but may require further investigation. Also, it remains true that we don't generally perceive whole objects at any given time, so memory must be involved in our construction of whole objects.
  • Srap Tasmaner
    4.6k
    Since we are not conscious of the process of input to the sensorium, I would agree that phenomena (defined as recognizable sense objects) are the output, so it seems we agree on that.Janus

    Cool.

    And if we reserve the word phenomena for what we can possibly be aware of, then objects are constitutive of phenomena, agreed?

    All this business about views — I understand what you're getting at, I just want to be clear that views are not phenomena. Views belong to the perceptual system, which offers to our awareness fully assembled phenomena complete with whole objects. (Ambiguity about how to classify an object makes no difference; it's an object, whatever it turns out to be.)

    As you say, there may corner cases, things not working as designed, and the perceptual system throws us something abnormal, but for the most part it's medium-sized dry goods.

    Are we still agreed?
  • Janus
    15.6k
    Are we still agreed?Srap Tasmaner

    :up: Yep, I have no argument with the idea that phenomena are, at least for the most part, objects.
  • Mww
    4.6k
    I thought the origin of the proofs themselves, being in the understanding, would be a priori…..Bob Ross

    The a priori/a posteriori distinction is determined by the what, not by the where. While understanding creates it own objects, re: numbers, a priori, proofs by means of them would be impossible if they were not made into real objects in the world.

    You can think all day long it takes three lines to enclose a space, but you’re not going to prove it with apodeitic mathematical certainty, unless you physically draw three real lines in a relation to each other corresponding to the image representing your thinking.

    ….we cannot know a priori the mathematical relations of objects a prioriBob Ross

    Correct. Relations of objects makes explicit experience, which is always and only determinable a posteriori.

    ……math is not a priori in the sense of being a part of our construction, via the understanding, of the world around us.Bob Ross

    Agreed, not part of our construction of the world, which begins with phenomena, whereas mathematics ends with them. In the former objects are given to us, in the latter objects are given by us.

    It isn’t that the possible worlds exist but, rather, that under one’s metaphysical commitments there is an existence with the potency to actualize the thing, and as such the thing is considered metaphysically possible.Bob Ross

    Sure, but so what? For me, a thing I have yet to experience is already metaphysically possible, simply because it is conceivable as a thing, or a manifold of things, such as a world of things. You’re saying a thing is metaphysically possible insofar as some existence with the potency to actualization some possible thing hasn’t done it yet, which is tantamount to a non-natural causality.

    Now, I accept the transcendental conception of a non-natural causality, but not with respect to the actualization of metaphysically possible things.
    ————

    (Paraphrased for simplicity)

    For example, under transcendental idealism I don’t think you can claim: there are other minds; that you have a mind; that you have representative faculties; that objects persist in their existences even when you are not perceiving them, etc……Bob Ross

    Transcendental philosophy is a speculative methodology. It doesn’t work by claims, which imply possible truths, but by internal logical consistency in the unity of abstract conceptions, same as yours.

    On the benefit of analytic idealism:

    I think a more plausible explanation and account of reality.Bob Ross

    Perhaps, but not more knowledge. So we have between us, one philosophy which demonstrates that some knowledge is impossible given this set of conditions, and another philosophy which demonstrates that the former impossible knowledge really isn’t, given a different set of conditions, which in effect, only demonstrates another form of impossible knowledge.

    Idealism, in whichever denomination, is always predicated on a subject that cognizes in accordance with a system contained in the form of his intellect. I rather think your idealism has to do with the cognitions, whereas my idealism has to do with the system proper; yours concerns what is thought about, mine with thought itself. Yours is limitless, mine self-limiting.

    When considering the pros and cons of each, parsimony should be the rule.
  • Bob Ross
    1.2k


    Hello Janus,

    I'm not becoming hostile, just impatient. I just don't believe that you are grasping what is meant by things in themselves. So, I am not going to deal with or respond to anything other than that one point at this juncture.

    Fair enough. Since we began this discussion as a conversation about Kantianism, I am going to assume you mean to be explaining it in terms thereof—unless you specify otherwise. In other words, I am going to assume by “thing-in-itself” you are referring to it in the Kantian sense: please let me know if you are a neo-Kantian or something else.

    The idea of things-in-themselves is not meant to be interpreted as claiming that there are things just like those that are perceived that exist independently of human perception; the "thing" in there is a kind of placeholder for some unknowable X

    I agree. Kant calls it a purely negative conception.

    So, object permanence cannot reasonably be thought to apply to things in themselves

    I agree.

    except in the sense that whatever it is that appears to us as invariant objects does so reliably, which suggests, but doesn't prove, that the in itself is invariant

    And here’s the problem: you can’t say that things-in-themselves cannot be thought of as knowably having object permanence and then turn around and say that the phenomena suggests that the things-in-themselves have object permanence. The phenomena do not suggest anything about the things-in-themselves under Kantianism. Period.

    In truth, we never perceive whole objects, but only views of them from different perspectives, so we construct the notion of whole objects from the various views (and feels) we have of them, and the fact that we can act on them, and the whole picture of a world of objects of more or less invariance is woven together with remarkable consistency by the brain/ mind.

    Would you say that we, then, get indirect knowledge of the things-in-themselves? I think that none of the above (that you said) is compatible with Kantianism, but I personally agree with you. Kant argues adamantly that we have absolutely no clue what the things-in-themselves are—not even a reverse engineering of the phenomena. See:

    We have intended, then, to say that all our intuition is nothing but the representation of phenomena; that the things we intuite, are not in themselves the same as our representations of them in intuition, nor are their relations in themselves so constituted as they appear to us; and that if we take away the subject, or even only the subjective constitution of our senses in general, then not only the nature and relations of objects in space and time, but even space and time themselves disappear; and that these, as phenomena, cannot exist in themselves, but only in us – CPR

    Perhaps you can be a neo-Kantian, but you are clearly contradicting Kant here.

    Part of this picture consists in the idea of object invariance; this idea is inevitable, even animal behavior shows that they expect objects not to simply disappear when they can't be seen. I observe this when I throw the ball for my dog and it inadvertently goes into the long grass; he never stops searching for it until he finds it showing that he expects it to be there somewhere and not to have simply disappeared.

    Janus, you are conceding here that you can, at the very least, get at what is suggested of the things-in-themselves via the phenomena, which is clearly not compatible with Kantianism (in its original formulation). I personally agree with you, but then you can’t turn around and claim, like a Kantian would (which was my whole point originally with Mww), that we can’t do metaphysics beyond transcendental philosophy. Your argument for object invariance here is exactly that: a metaphysical claim pertaining to the things-in-themselves.

    It is exactly what you are arguing against right here:

    So, if anyone says that they think this or that metaphysical explanation is the most plausible, that really only speaks to their own personal preferences. That, in short, is all I've been arguing for.

    So, I haven't been arguing that it is provable that the in itself is invariant or that phenomenal objects are "permanent", but that object permanence is the inference to the best explanation in the empirical context, and that regarding noumenal invariance we really have no idea how to assess which explanation would be the more plausible because we have nothing to compare any explanation with

    “noumenal invariance” and “object invariance” are the same thing: they are both a metaphysical claim about the same things-in-themselves. By definition (of “object invariance”), we are talking about whatever persists beyond your phenomenal experience and is thusly non-phenomenal (i.e., noumenal).

    Bob
  • Bob Ross
    1.2k


    Hello Mww,

    You can think all day long it takes three lines to enclose a space, but you’re not going to prove it with apodeitic mathematical certainty, unless you physically draw three real lines in a relation to each other corresponding to the image representing your thinking.

    But the image I draw won’t necessarily be accurate and thusly will not prove it in itself. For example, take a circle: the circumference is 2πr. There is no circle, Mww, that you can draw for me that will actually have a circumference of exactly 2πr. The proof is within the abstractions and the drawings are approximations thereof and, consequently, afford no such apodeitic mathematical certainty on there own.

    Agreed, not part of our construction of the world, which begins with phenomena, whereas mathematics ends with them

    I think you may be saying something different than me while still agreeing on mathematical anti-realism: you seem to be saying that math is ingrained into the a priori construction of the phenomenal world, whereas I am not even granting that much.

    I think that our mathematical formulas are good estimations of our qualitative experience, which is necessarily not quantitative—so there’s no math in it. Reality is fundamentally a clash of wills.

    For me, a thing I have yet to experience is already metaphysically possible, simply because it is conceivable as a thing, or a manifold of things, such as a world of things

    But not all conceivable things are metaphysically possible. For example, under your metaphysics the understanding is not a phenomena: so it is metaphysically impossible for the understanding to be a phenomena. However, we can both imagine a world where the understanding is phenomena. But that doesn’t matter because, under your metaphysical view, that’s impossible because there is nothing you believe than actualize that potential.

    You’re saying a thing is metaphysically possible insofar as some existence with the potency to actualization some possible thing hasn’t done it yet, which is tantamount to a non-natural causality.

    I don’t see how this follows. Something can have the potential to be actualized metaphysically and be perfectly natural. For example, under my view, it is actually and metaphysically possible for the ball at the top of the hill to fall to the ground because I belief the world has to offer such things that could actualize it.

    Now, I accept the transcendental conception of a non-natural causality, but not with respect to the actualization of metaphysically possible things.

    I didn’t follow this point: what do you mean by the “non-natural causality” in transcendental philosophy? And what do you mean in terms of its contradistinction to the “actualization of metaphysically possible things”?

    For example, under transcendental idealism I don’t think you can claim: there are other minds; that you have a mind; that you have representative faculties; that objects persist in their existences even when you are not perceiving them, etc…… — Bob Ross

    Transcendental philosophy is a speculative methodology. It doesn’t work by claims, which imply possible truths, but by internal logical consistency in the unity of abstract conceptions, same as yours.

    I am failing to see how this was a response to my objections that you quoted. What do you mean by “it doesn’t work by claims”?

    Perhaps, but not more knowledge. So we have between us, one philosophy which demonstrates that some knowledge is impossible given this set of conditions, and another philosophy which demonstrates that the former impossible knowledge really isn’t, given a different set of conditions, which in effect, only demonstrates another form of impossible knowledge.

    What is the other form of impossible knowledge that my theory conceives?

    Idealism, in whichever denomination, is always predicated on a subject that cognizes in accordance with a system contained in the form of his intellect

    Correct.

    I rather think your idealism has to do with the cognitions, whereas my idealism has to do with the system proper;

    I think from your perspective I am “skipping over” the representative faculty of my mind which is supposed to the ultimate metaphysical dead-end. Whereas, I think we can extend the same metaphysical inquiries to all of reality.

    yours concerns what is thought about, mine with thought itself.

    I can agree with this to a certain extent; but I also hold that our minds are representative faculties—however, I don’t think it is cogent to claim that we can only go that far. Likewise, as a side note, I am not thoroughly convinced of Kant’s 12 categories: I appreciated Schopenhauer’s idea of the understanding as simply the PSR of becoming better.

    Yours is limitless, mine self-limiting.

    Fair.

    When considering the pros and cons of each, parsimony should be the rule.

    Parsimony, internal/external coherence, empirical adequacy, intuitions, explanatory power, reliability, and credence. I would say.

    Bob
  • Janus
    15.6k
    And here’s the problem: you can’t say that things-in-themselves cannot be thought of as knowably having object permanence and then turn around and say that the phenomena suggests that the things-in-themselves have object permanence. The phenomena do not suggest anything about the things-in-themselves under Kantianism. Period.Bob Ross

    If things-in-themselves are responsible for producing the phenomenal things, and the phenomenal things are reliably invariant (to varying degrees according to the phenomena under consideration, of course) then we can say that things in themselves reliably give rise to invariant phenomena. That doesn't say anything about the things in themselves being invariant in themselves, though.

    except in the sense that whatever it is that appears to us as invariant objects does so reliably, which suggests, but doesn't prove, that the in itself is invariant

    And here’s the problem: you can’t say that things-in-themselves cannot be thought of as knowably having object permanence and then turn around and say that the phenomena suggests that the things-in-themselves have object permanence. The phenomena do not suggest anything about the things-in-themselves under Kantianism. Period.
    Bob Ross

    What I meant there is the same as what I said above; we have no warrant for saying that things-in-themselves are invariant in themselves, but we do know that they are invariant in the sense that they reliably produce invariant phenomena. In positing things-in-themselves as being the things that give rise to the appearance of phenomenal things I'd say Kant must be committed to that much.

    Now I admit that there is a tension here in the Kantian idea that we know absolutely nothing about things-in-themselves, but I don't think it amounts to an outright inconsistency.

    However, I am not a Kant scholar, so I can't say whether Kant addresses this area of potential tension or passes it over. In any case, the point is that if all we know about things in themselves is that they invariably produce invariant phenomena, then it remains that we know nothing at all about what they are in themselves.

    Would you say that we, then, get indirect knowledge of the things-in-themselves? I think that none of the above (that you said) is compatible with Kantianism, but I personally agree with you. Kant argues adamantly that we have absolutely no clue what the things-in-themselves are—not even a reverse engineering of the phenomena. See:

    We have intended, then, to say that all our intuition is nothing but the representation of phenomena; that the things we intuite, are not in themselves the same as our representations of them in intuition, nor are their relations in themselves so constituted as they appear to us; and that if we take away the subject, or even only the subjective constitution of our senses in general, then not only the nature and relations of objects in space and time, but even space and time themselves disappear; and that these, as phenomena, cannot exist in themselves, but only in us – CPR

    Perhaps you can be a neo-Kantian, but you are clearly contradicting Kant here.
    Bob Ross

    If "the nature and relations of objects in space and time" and space and time themselves are human representations, human perceptions, then it would seem to follow that these cannot exist apart from human experience. Look at it another way: what could it even mean to say that the objects and relations and the very modalities themselves of human experience existed absent human experience? All he is saying is that the phenomena of perception cannot exist absent perception, and that seems right, doesn't it?

    Janus, you are conceding here that you can, at the very least, get at what is suggested of the things-in-themselves via the phenomena, which is clearly not compatible with Kantianism (in its original formulation). I personally agree with you, but then you can’t turn around and claim, like a Kantian would (which was my whole point originally with Mww), that we can’t do metaphysics beyond transcendental philosophy. Your argument for object invariance here is exactly that: a metaphysical claim pertaining to the things-in-themselves.Bob Ross

    No, you're misunderstanding again. I'm only talking about the natural expectations of the dog that objects don't simply disappear when not being perceived. And we know they don't because they are generally always there where they were last time we looked. But this speaks only to the phenomenal objects, it says nothing about the things-in-themselves other than that they must be thought to somehow invariably produce this object permanence.

    “noumenal invariance” and “object invariance” are the same thing: they are both a metaphysical claim about the same things-in-themselves. By definition (of “object invariance”), we are talking about whatever persists beyond your phenomenal experience and is thusly non-phenomenal (i.e., noumenal).Bob Ross

    No, they are not the same thing, Bob. It might seem inconceivable to us that something could produce a world of differentiated and diversely invariant objects without being differentiated and invariant in itself, but it doesn't follow that we therefore know that the in itself must be differentiated and invariant. We might think that to be the most plausible explanation, but quantum physics might make us think twice about that (even though quantum physics only talks about how things are as they appear to us, we might think it is the closest we can get to the in itself, and it presents us with counter-intuitive and paradoxical pictures).




    .
  • Mww
    4.6k
    ”yours concerns what is thought about, mine with thought itself.”
    -Mww

    I can agree with this to a certain extent; but I also hold that our minds are representative faculties—however, I don’t think it is cogent to claim that we can only go that far.
    Bob Ross

    One of the subtleties of metaphysics in general, is the recognition that only through reason can reason be examined, from which follows that all that is reasoned about is predicated on what is reason is. This is, of course, the epitome of circularity, and because it is inevitable, it best be kept to a minimum. No one has admitted to having sufficient explanation for how we arrive at representations, even while many philosophize concerning what they do in a speculative theory, justifying their inclusions in it. So saying, to posit an additional representational faculty, doing what it does and we not being able to say how it does what it does, stretches circularity beyond what couldn’t be explained beforehand.
    —————

    What do you mean by “it doesn’t work by claims”?Bob Ross

    I mean you are correct, in that there are things, such as those you listed, that I have no warrant to claim, either as fact objectively, or as irreducible truth subjectively, which is exactly the conditions under which transcendental philosophy is to be understood.
    —————

    ”…..only demonstrates another form of impossible knowledge.”
    -Mww

    What is the other form of impossible knowledge that my theory conceives?
    Bob Ross

    You hold that knowledge of the nature of the thing-in-itself is knowable, which is knowledge I hold as impossible, yet you hold with the mind as a representational faculty, which is something impossible to know without the antecedent knowledge there is a mind, and, the nature of it is such that it has representational capabilities.
    —————

    …..under my view, it is actually and metaphysically possible for the ball at the top of the hill to fall to the ground because I belief the world has to offer such things that could actualize it.Bob Ross

    That the world offers (or withholds) is just another way of saying there’s a set of relations between the world and its objects, one set of relations and this happens, another set of relations and that happens which is the same as this doesn’t happen, all from the perspective of an intelligence capable of characterizing relations.
    —————

    But not all conceivable things are metaphysically possible.Bob Ross

    If conception is itself a metaphysical function, and if possibility is a metaphysical condition, then whatever is conceivable must be metaphysically possible.
    —————

    ….you can’t turn around and claim, like a Kantian would (which was my whole point originally with Mww), that we can’t do metaphysics beyond transcendental philosophy.Bob Ross

    No matter what was turned around from, or by whom, I never said nor hinted there is no metaphysics beyond transcendental philosophy, or that all metaphysics is necessarily predicated on transcendental philosophy’s critical method.

    “…. (That for) which, with all its preliminaries, has for its especial object the solution of these problems (of pre reason) is named metaphysics….”

    One can attempt to solves pure reason’s problems, including the one of singular importance, any way he wishes, depending on the preliminaries he uses.

    Perhaps you might be so kind as to reiterate what your whole point originally was, with respect to what you said there.

    Do you have an idea as to why your system is called analytic idealism, insofar as it is a metaphysical doctrine?
  • Bob Ross
    1.2k


    Hello Janus,

    If things-in-themselves are responsible for producing the phenomenal things, and the phenomenal things are reliably invariant (to varying degrees according to the phenomena under consideration, of course) then we can say that things in themselves reliably give rise to invariant phenomena. That doesn't say anything about the things in themselves being invariant in themselves, though.

    I think it does, because the only way a thing a representation can be invariant is if either (1) the mind of which it is produced simply fabricates it as such, or (2) the object of which it is representing (which is a thing-in-itself) is invariant. There’s no other options by my lights.

    What I meant there is the same as what I said above; we have no warrant for saying that things-in-themselves are invariant in themselves, but we do know that they are invariant in the sense that they reliably produce invariant phenomena

    I understand what you are saying here, but this is predicated on the idea that there is another option, plausibly, other than #1 and #2 above.

    In positing things-in-themselves as being the things that give rise to the appearance of phenomenal things I'd say Kant must be committed to that much.

    I don’t think so because I think #1 and #2 are the only options, and Kant denies #2; so #1 is the only thing plausibly left. Otherwise, Kant (and you) would be saying that the invariance of the phenomena are not fabrications of one’s mind and they aren’t a reflection of the objects themselves—but if the latter is true then they their invariance must be due to the mind.

    Now I admit that there is a tension here in the Kantian idea that we know absolutely nothing about things-in-themselves, but I don't think it amounts to an outright inconsistency.

    I think, given what I said above, it is an inconsistency. Either the mind’s representative faculty cause the invariance, or the things-in-themselves which are being represented do: there’s no third option. So to make a claim like “we can know phenomenal invariance but nothing about the things-in-themselves”, to me, is claiming a third option that can’t exist.

    If "the nature and relations of objects in space and time" and space and time themselves are human representations, human perceptions, then it would seem to follow that these cannot exist apart from human experience.

    By my lights, if space and time are pure intuitions, then the phenomenal permanence must be in our minds representative faculties (and not the things-in-themselves)—unless one wants to posit a second outer space and time beyond our minds. If this is the case, though, then there’s no reason to believe that there are outer objects (things-in-themselves) in any sense that we perceive them, and thusly, that they have permanence (other than our minds creating them fictitiously as permanent).

    All he is saying is that the phenomena of perception cannot exist absent perception, and that seems right, doesn't it?

    I don’t think that’s just what he is saying: our perceptions are all we can know according to him; they are perceptions of X, and X is never knowable. Thusly, we can’t even know that the things-in-themselves have object permanence, which I think is highly unparsimonious.

    I'm only talking about the natural expectations of the dog that objects don't simply disappear when not being perceived

    If don’t think you can know that the things-in-themselves produce the permanence (which you can’t if Kant is right), then the only other option is that the mind creates it as a matter of fiction—which would mean that it isn’t there when you aren’t perceiving it.

    It might seem inconceivable to us that something could produce a world of differentiated and diversely invariant objects without being differentiated and invariant in itself, but it doesn't follow that we therefore know that the in itself must be differentiated and invariant

    To me, it does. Otherwise, you are invoking magic, which is always unparsimonious. (1) the mind of which it is produced simply fabricates it as such, or (2) the object of which it is representing (which is a thing-in-itself) is invariant.

    Likewise, to say that the phenomenal invariance is caused by something which itself is not invariant is to say that it is a mere appearance and thusly there isn’t really object permanence.

    Think of it this way: if the things-in-themselves are not persisting when you are not representing them but yet your phenomenal experience of them is that they are persistent, then the only option available to you is that the mind is fabricating it and thusly the persistence is an illusion: the object doesn’t persist when you are not looking at it because that persistence is a feature of your mind.

    We might think that to be the most plausible explanation, but quantum physics might make us think twice about that

    I don’t see how quantum mechanics is helping your case: could you please elaborate?

    Bob
  • Bob Ross
    1.2k


    Hello Mww,

    One of the subtleties of metaphysics in general, is the recognition that only through reason can reason be examined, from which follows that all that is reasoned about is predicated on what is reason is. This is, of course, the epitome of circularity, and because it is inevitable, it best be kept to a minimum. No one has admitted to having sufficient explanation for how we arrive at representations, even while many philosophize concerning what they do in a speculative theory, justifying their inclusions in it. So saying, to posit an additional representational faculty, doing what it does and we not being able to say how it does what it does, stretches circularity beyond what couldn’t be explained beforehand.

    This is fair.

    I mean you are correct, in that there are things, such as those you listed, that I have no warrant to claim, either as fact objectively, or as irreducible truth subjectivity, which is exactly the conditions under which transcendental philosophy is to be understood.

    Oh I see. Are you agreeing with me then that:

    [/quote]For example, under transcendental idealism I don’t think you can claim: there are other minds; that you have a mind; that you have representative faculties; that objects persist in their existences even when you are not perceiving them, etc[/quote]

    yet you hold with the mind as a representational faculty, which is something impossible to know without the antecedent knowledge there is a mind, and, the nature of it is such that it has representational capabilities.

    I am not following which part of this is impossible knowledge (other than your claim that we cannot know the things-in-themselves). Yes, I think we can know that there are minds that represent the world around to themselves: what is impossible (in terms of knowledge) about that?

    If conception is itself a metaphysical function, and if possibility is a metaphysical condition, then whatever is conceivable must be metaphysically possible.

    Why would conceptions be a metaphysical function?

    No matter what was turned around from, or by whom, I never said nor hinted there is no metaphysics beyond transcendental philosophy, or that all metaphysics is necessarily predicated on transcendental philosophy’s critical method.

    I apologize: I must have misunderstood you. I thought you were claiming that we cannot perform valid metaphysics beyond transcendental philosophy—as we cannot know the things-in-themselves. Is that incorrect?

    One can attempt to solves pure reason’s problems, including the one of singular importance, any way he wishes, depending on the preliminaries he uses.

    I wasn’t saying that you were defining metaphysics as only transcendental philosophy but, rather, that you would claim we cannot do metaphysics beyond transcendental investigations because that is impossible knowledge for you.

    Perhaps you might be so kind as to reiterate what your whole point originally was, with respect to what you said there.

    With respect to what was said there, I was saying responding to:

    except in the sense that whatever it is that appears to us as invariant objects does so reliably, which suggests, but doesn't prove, that the in itself is invariant – Janus

    And here’s the problem: you can’t say that things-in-themselves cannot be thought of as knowably having object permanence and then turn around and say that the phenomena suggests that the things-in-themselves have object permanence. The phenomena do not suggest anything about the things-in-themselves under Kantianism. Period.

    And the part you quoted was here (in its full context):

    Janus, you are conceding here that you can, at the very least, get at what is suggested of the things-in-themselves via the phenomena, which is clearly not compatible with Kantianism (in its original formulation). I personally agree with you, but then you can’t turn around and claim, like a Kantian would (which was my whole point originally with Mww), that we can’t do metaphysics beyond transcendental philosophy. Your argument for object invariance here is exactly that: a metaphysical claim pertaining to the things-in-themselves.

    I am discussing with Janus about whether one, as a Kantian, can claim there is object permanence. Janus believes so (under the conditions they are explaining), and I say no. When I invoked you I was just tying in our conversation that I think we can go beyond transcendental metaphysics, and I think you would say we can’t (because knowledge of the things-in-themselves) is impossible. I think this is a self-undermining under Kantianism (for many reasons that we have already expounded).

    I apologize if I misrepresented you there, please correct me where I am wrong.

    Do you have an idea as to why your system is called analytic idealism, insofar as it is a metaphysical doctrine?

    It is originally called ‘analytic’ idealism because it is formulated under the Analytic school of philosophy, but I like it, beyond that personally, due to my definition of objectivity: so I prefer saying “analytic idealism” over “objective idealism” because I don’t like misusing the term ‘objective’ in that manner. Technically, it is a form of ‘objective idealism’ as formulated by Kastrup.

    Bob
  • Janus
    15.6k
    I think it does, because the only way a thing a representation can be invariant is if either (1) the mind of which it is produced simply fabricates it as such, or (2) the object of which it is representing (which is a thing-in-itself) is invariant. There’s no other options by my lights.Bob Ross

    The way I see it, everything that is outside of our conscious experience including what we don't know about ourselves is what we refer to apophatically as the noumenal. We know that we experience invariant objects in invariant environments, and all we can do regarding what is beyond our knowledge and experience is try to imagine what that noumenal being is; the problem with that being that all our imaginings are in terms of what we know of the experienced world.

    So, there may be no other option for you than the two you outlined there, but that says everything about you and nothing about the noumenal as I see it. For me the other option, apart from the inevitably aporetic imaginable ones, is simply that we cannot know, and have no way of comparing our conjectures about the noumenal with anything in order to assess their plausibility.

    I think, given what I said above, it is an inconsistency. Either the mind’s representative faculty cause the invariance, or the things-in-themselves which are being represented do: there’s no third option. So to make a claim like “we can know phenomenal invariance but nothing about the things-in-themselves”, to me, is claiming a third option that can’t exist.Bob Ross

    Our minds, by themselves, being as far as we can tell, unconnected with one another cannot plausibly be thought to cause invariances which are common to all percipients. So the thing-in-itself must cause the invariance; that is it must be mind-independent, in the sense of being independent of any and all individual minds (which are the only minds we know about). But, as I have said, it doesn't follow that the thing in itself is invariant, other than in its capacity to cause experienced invariance. Perhaps, it might be thought, as per some recent theses in QM, that everything is really one thing: an entangled unity; but then we have the problem of how it could produce experienced diversity.

    Face it Bob, whatever we imagine about the noumenal, we are always going to run up against aporias. Personally, I think this is because all our categories of thought are based on our dualistic mode of thinking, and those categories just don't apply to the noumenal. If this is right, the noumenal can never be experienced or understood and our speculations are merely "pouring from the empty into the void". All that said, if you enjoy speculating then go for it, but I'd be wary of taking those speculations seriously.

    I'm going to leave it there, Bob: it's been fun, but we are inevitably going to continue to go around in circles. Thanks for the conversation.
  • Mww
    4.6k
    Are you agreeing with me then that:
    Bob Ross
    For example, under transcendental idealism I don’t think you can claim: there are other minds; that you have a mind; that you have representative faculties; that objects persist in their existences even when you are not perceiving them, etc[/quote][/quote]

    I don’t think there’s sufficient warrant to claim there are other minds in any case, but it is nonetheless reasonable to suppose there are.

    I recognize the ubiquity of the conventional use of the word, but I personally don’t hold with minds as something a human being has. I consider it justified to substitute reason for mind anywhere in a dialectic without detriment to it, given the fact it is impossible to deny, all else being equal, that every human is a thinking subject. On the other hand, I am perfectly aware I am a thinking subject, which authorizes me to claim reason for myself, and that beyond all doubt.

    I don’t claim representational faculties, but affirm the predicates of a speculative philosophy that presupposes them. They are explanatory devices in a discipline where the empirical support of experience is absent.

    That objects do not persist in the absence of perception leads to irreconcilable contradictions, which suggests the claim as to whether or not they do, is an irrational inquiry. The logical consistency inherent in human intelligence demands only that which can be said about the relation a perceived object has with me, as opposed to the relation an unperceived object has with me, in either case the object’s existence is presupposed, from which follows the ontology of the object should never be in question.

    The absurdity resides in the notion that if non-perception implies non-existence, then my perception is necessary existential causality itself. But it is absolutely impossible for me to cause the existence of whatever I wish to perceive, as well as to not perceive that of which I have no wish whatsoever, which makes explicit the only existences I could possibly be the causality of, is that which was already caused otherwise, which is all my perceptions could ever tell me anyway.

    Then there is time. If I am the cause of an object’s existence merely from my perception of it, then the time of my perception is identical to the time of the object’s existence, which is the same as my having attributed to that object the property of time. But time, as well as space, can never be assigned as a property, therefore the time or space of the object’s existence cannot be an attribution of mine, which makes explicit the time and space of an unperceived object is a duration of a time in general and a position in a space in general, which for me is the same as any time and space in general, which is not necessarily the particular time and space of the object of my perception.
    ————-

    Yes, I think we can know that there are minds that represent the world around to themselves: what is impossible (in terms of knowledge) about that?Bob Ross

    In order to know a thing in the strictest sense, it must manifest as an experience. What is impossible (in terms of knowledge) about that, is that minds of any form are never going to manifest as an experience. You alternative is to not conceive minds that represent the world, as things, or, to characterize knowledge as something other than that which manifests as experience.

    Complicated further but the annexation of “to themselves”. If it is the case minds that represent are not met with the criteria for knowledge, then a mind that represents to itself is unintelligible.

    So the question remains…..how would such knowledge be possible? How is it that you think that which the judgement represents, can be known?
    —————

    I thought you were claiming that we cannot perform valid metaphysics beyond transcendental philosophy—as we cannot know the things-in-themselves. Is that incorrect?Bob Ross

    That we cannot know the thing-in-itself has nothing to do with metaphysics. Metaphysics proper concerns itself with solutions to the problems pure reason brings upon itself, of which the thing-in-itself is not one. In fact, the thing-in-itself shouldn’t be a problem in any case, under the purview of the theory from which it originates, re: transcendental philosophy. There are those that make a problem out of it merely by altering judgement of its original conception and its subsequent derivatives, which just culminates in the installation of a different philosophy.

    You mentioned good vs bad metaphysics a few pages ago. I didn’t think that a worthy distinction then, and I don’t think valid/invalid metaphysics to be any better now. Good vs bad logic in conjunction with experience or possible experience, for whatever metaphysics, has better service.
    ————

    It is originally called ‘analytic’ idealism because it is formulated under the Analytic school of philosophyBob Ross

    Ahhhh…that’s it? Transcendental idealism shifted the entire idealistic paradigm, so I figured that which attempts to shift it again, would shift from that. There is a short missive in CPR which sets the ground for its doctrine, which says metaphysics is predicated necessarily on the possibility of synthetic a priori cognitions, then goes about proving there are such things which validates the ground initially set as a premise. That to which synthetic cognitions are juxtaposed, are analytic, so….I just figured the new style of idealism wanted to be grounded in pure analytic cognitions, which are mere tautologies necessarily true in themselves, which, of course, a universal mind would have to be, re: self-evident. I mean….what would there be to synthesize to a universal, which makes synthetic cognitions with respect to that concept, impossible, which means that condition must itself be, well…..analytic.

    But now….never mind. Just flew into my head, in keeping with what some philosophers historically do with themselves: take what one said, change this and that a little bit, present it as something new and different.
  • Bob Ross
    1.2k


    I'm going to leave it there, Bob: it's been fun, but we are inevitably going to continue to go around in circles. Thanks for the conversation.

    Fair enough my friend! I enjoyed our conversation and look forward to future ones!

    Have a wonderful day!
    Bob
  • Bob Ross
    1.2k


    Hello Mww,

    I don’t think there’s sufficient warrant to claim there are other minds in any case, but it is nonetheless reasonable to suppose there are.

    Why would it be reasonable if you cannot know anything about the things-in-themselves, which would include other minds? Wouldn’t it be most reasonable to be an epistemic solipsist?

    I recognize the ubiquity of the conventional use of the word, but I personally don’t hold with minds as something a human being has. I consider it justified to substitute reason for mind anywhere in a dialectic without detriment to it, given the fact it is impossible to deny, all else being equal, that every human is a thinking subject. On the other hand, I am perfectly aware I am a thinking subject, which authorizes me to claim reason for myself, and that beyond all doubt.

    But there are things about you as a mind you cannot prove of others without venturing into metaphysical claims about the things-in-themselves. Yes, we all reason, but that’s really not what a mind is in the context of solipsism. It just seems like an evasion (inadvertently) of the real issue I am trying to address here to say that ‘mind’ is merely ‘reasoning’.

    Likewise, you can’t prove, even if that is the case that we all reason, that ‘we’ are the ‘ones reasoning’. Do you agree with me on that?

    The absurdity resides in the notion that if non-perception implies non-existence, then my perception is necessary existential causality itself. But it is absolutely impossible for me to cause the existence of whatever I wish to perceive, as well as to not perceive that of which I have no wish whatsoever, which makes explicit the only existences I could possibly be the causality of, is that which was already caused otherwise, which is all my perceptions could ever tell me anyway.

    Yes, but if you can’t know anything about the things-in-themselves, then you can’t know that it is absurd for your mind to be producing it all.

    Then there is time. If I am the cause of an object’s existence merely from my perception of it, then the time of my perception is identical to the time of the object’s existence, which is the same as my having attributed to that object the property of time. But time, as well as space, can never be assigned as a property, therefore the time or space of the object’s existence cannot be an attribution of mine

    Time and space aren’t properties of objects per se, but you are, under transcendental idealism, producing them under space and time. Saying that the objects only exist in your perception is just to say that there no corresponding object beyond those forms of space and time: it isn’t to say that the objects themselves can be attributed the property of time in the same manner as the property of being red.

    In order to know a thing in the strictest sense, it must manifest as an experience. What is impossible (in terms of knowledge) about that, is that minds of any form are never going to manifest as an experience.

    It can agree with this, as a matter of semantics, if you are saying that possible knowledge is that which one experiences; but then this just pushes the question back: why can’t we say that possible knowledge goes beyond our experiences?

    Also, as a side note, wouldn’t it be impossible to know that, for example, your mind uses pure conceptions of the understanding to produce the world if we are defining possible knowledge as only that which we experience? Because we definitely don’t experience that.

    how would such knowledge be possible? How is it that you think that which the judgement represents, can be known?

    Because we can tell that our perception of the world is dictated by our representative faculties. For example, there are color blind people: this is due to their minds representing the world with disabled functionality.

    That we cannot know the thing-in-itself has nothing to do with metaphysics. Metaphysics proper concerns itself with solutions to the problems pure reason brings upon itself, of which the thing-in-itself is not one.

    It most certainly is. Metaphysics is about understanding that which is beyond all possibility of experience, and that includes transcendental philosophy.

    Things-in-themselves are beyond the possibility of all experience.

    Good vs bad logic in conjunction with experience or possible experience, for whatever metaphysics, has better service.

    Metaphysics predicated solely on logic is bad metaphysics. That only gets them to a logically consistent view. Parsinomy, coherence, empirical adequacy, and intuitions are just some examples of pertinent non-logical factors.

    Ahhhh…that’s it? Transcendental idealism shifted the entire idealistic paradigm, so I figured that which attempts to shift it again, would shift from that.

    Analytic Idealism, I would say, is just pure ontolotical idealism; whereas transcendental idealism is really only epistemic idealism—it isn’t idealism in the ontological sense. So I wouldn’t say analytic idealism has shifted the paradigm again, this is an old view starting with schopenhauer, plato, etc.

    There is a short missive in CPR which sets the ground for its doctrine, which says metaphysics is predicated necessarily on the possibility of synthetic a priori cognitions, then goes about proving there are such things which validates the ground initially set as a premise. That to which synthetic cognitions are juxtaposed, are analytic, so….I just figured the new style of idealism wanted to be grounded in pure analytic cognitions, which are mere tautologies necessarily true in themselves, which, of course, a universal mind would have to be, re: self-evident

    I am not sure I completely followed this, but the idea would be to say simply that the universal mind is the best explanation for the world that is given to us. Also, I don’t think it uses Kant’s synthetic vs. analytic distinction; or if it does then it denies that the reality which we can know is purely synthetic of our minds.

    Bob
  • Mww
    4.6k
    ”I don’t think there’s sufficient warrant to claim there are other minds in any case, but it is nonetheless reasonable to suppose there are.”
    -Mww

    Why would it be reasonable if you cannot know anything about the things-in-themselves, which would include other minds?
    Bob Ross

    Things-in-themselves concerns things. Minds are not things. Things-in-themselves do not include minds.

    But there are things about you as a mind you cannot prove of others without venturing into metaphysical claims about the things-in-themselves.Bob Ross

    I am not a mind; I am a conscious intelligence, a thinking subject, which is certainly a metaphysical claim. Notice the conspicuous lack of mention for the thing-in-itself. My body is never absent from my representational faculties, insofar as they are contained in it, thus is always a thing and never a thing-in-itself.

    It just seems like an evasion (inadvertently) of the real issue I am trying to address here to say that ‘mind’ is merely ‘reasoning’.Bob Ross

    I didn’t say mind was merely reasoning. Such idea makes no sense to me. As well, I’m responding in kind to your verbatim comments, so if I’m evading it’s because I am not aware of what you’re trying to address.

    Likewise, you can’t prove, even if that is the case that we all reason, that ‘we’ are the ‘ones reasoning’. Do you agree with me on that?Bob Ross

    Sure. It is not impossible what I consider as thinking really isn’t, but is in fact merely the material complexity of my brain manifesting as the seeming of thought. So, what…..you’re trying to say that because it is not impossible for thinking to be other than it seems, the door is thereby left open for my thinking to be a manifestation of something even outside my own brain? Perhaps that’s no more than the exchange of not impossible regarding brains, for vanishingly improbable for external universal entity.
    —————

    Time and space aren’t properties of objects per se, but you are, under transcendental idealism, producing them under space and time.Bob Ross

    No. I am not producing objects. I am producing representations of them, and those under, or conditioned by, space and time.

    Saying that the objects only exist in your perception is just to say that there no corresponding object beyond those forms of space and timeBob Ross

    Sure, but no one has sufficient justification for saying objects only exist in perception, which makes the rest irrelevant.

    ”In order to know a thing in the strictest sense, it must manifest as an experience.”
    -Mww

    It can agree with this, as a matter of semantics, if you are saying that possible knowledge is that which one experiences……
    Bob Ross

    Semantics, huh? Why don’t we just agree that if you know a thing, you’ve experienced it.

    ……but then this just pushes the question back: why can’t we say that possible knowledge goes beyond our experiences?Bob Ross

    Why wouldn’t that be true? The truth of that doesn’t affect the premise that if a thing is known it must have been an experience, and doesn’t affect possible experience.

    Also, as a side note, wouldn’t it be impossible to know that, for example, your mind uses pure conceptions of the understanding to produce the world if we are defining possible knowledge as only that which we experience? Because we definitely don’t experience that.Bob Ross

    Of course. The categories are nothing but theoretical constructs. It is merely a logically consistent speculation that understanding relates pure conceptions to cognition of things. Pretty hard to experience a theory, right?
    ————-

    Because we can tell that our perception of the world is dictated by our representative faculties.Bob Ross

    Now, for me, this is exactly backwards. I mean…what comes first, the appearance of a thing, or the representation of it? Our understanding of the world is dictated by our representational faculties.

    Metaphysics is about understanding that which is beyond all possibility of experience, and that includes transcendental philosophy.Bob Ross

    Ehhhhh…..we just have different ideas of what entails metaphysics. While it may be fine to say it is understood for something to be beyond the possibility of all experience, it remains the case that understanding is not authorized to say what that something is, but only that the criteria for experience has not been met.

    Understanding cannot inform what things are not conditioned by the categories, but only informs regarding those that are. Without the categorical criteria, understanding can still conceive on its own, but mere conception is by no means sufficient causality, from which follows that understanding cannot determine the ground of experience on its own accord. Something else must intervene, in order for subsequent understanding to grant the invalidity of its conceptions….the error in its judgement……with respect to possible experience because of them.

    “…. By exposition, I mean the clear, though not detailed, representation of that which belongs to a conception; and an exposition is metaphysical when it contains that which represents the conception as given à priori.…”

    Yours wants the content of a conception as metaphysical, which is an exposition of it; mine wants that there are conceptions, including their content, not thought spontaneously as in understanding in conjunction with a synthesis of relations, but given complete in themselves from a pure a priori source. Reason is the only human faculty with the power to forward conceptions complete in themselves, which are called objects of reason, or, transcendental objects, and are not at all objects of experience.

    So, yes, it is understood there are things beyond experience, but metaphysics in relation to understanding is not how they are given.

    Things-in-themselves are beyond the possibility of all experience.Bob Ross

    And of course, the thing-in-itself is no more that a full-fledged, self-contained conception arising from…..that’s right……pure reason. A metaphysical conception understood within an empirical domain, but not given from it.
    ————-

    Analytic Idealism, I would say, is just pure ontolotical idealism; whereas transcendental idealism is really only epistemic idealismBob Ross

    That sounds reasonable to me, and also serves as a barrier for the compatibility of our respective philosophies. Which is fine, I don’t mind, but we’ll run out of things to discuss sooner rather than later.
  • Bob Ross
    1.2k


    Hello Mww,

    Things-in-themselves concerns things. Minds are not things. Things-in-themselves do not include minds.

    Are you a substance dualist? It sounds like you are saying there are minds which are of a mental substance and there are things-in-themselves which are a part of a physical substance. Otherwise, I do not know what you mean here.

    I am not a mind; I am a conscious intelligence, a thinking subject

    But, traditionally, a mind is a conscious intelligence—a thinking subject which has qualia.

    Notice the conspicuous lack of mention for the thing-in-itself. My body is never absent from my representational faculties, insofar as they are contained in it, thus is always a thing and never a thing-in-itself.

    I agree that the body is not a thing-in-itself, but the mind (or something else) must be. Even if the mind is not a ‘thing’ in the sense of being of a physical substance, it is a ‘thing-in-itself’ of a mental substance. ‘Thing’ here is being used more vaguely as a purely negative conception (like Kant used it). It could be a mental ‘thing’ or a physical ‘thing’.

    I didn’t say mind was merely reasoning.

    Sorry, I must have misunderstood then. What is a mind to you then?

    It is not impossible what I consider as thinking really isn’t, but is in fact merely the material complexity of my brain manifesting as the seeming of thought. So, what…..you’re trying to say that because it is not impossible for thinking to be other than it seems, the door is thereby left open for my thinking to be a manifestation of something even outside my own brain? Perhaps that’s no more than the exchange of not impossible regarding brains, for vanishingly improbable for external universal entity.

    I am saying that you can’t prove, because you think we cannot know anything about the things-in-themselves (even probabilistically speaking), that (1) other people have conscious experience and (2) that your own thoughts are associated with an ‘I’ which is beyond the phenomena.

    You can’t appeal to probability nor plausibility for #1 or #2 because you are saying we cannot know the things-in-themselves, and those claims are about them: even if it is about fundamentally mental ‘things’.

    Time and space aren’t properties of objects per se, but you are, under transcendental idealism, producing them under space and time. — Bob Ross

    No. I am not producing objects. I am producing representations of them, and those under, or conditioned by, space and time.

    Correct, I misspoke: I was saying that space and time are produced by your mind, not that the objects themselves are. They are produced by your mind because they are the pure forms of intuition that your mind uses to represent objects.

    Saying that the objects only exist in your perception is just to say that there no corresponding object beyond those forms of space and time — Bob Ross

    Sure, but no one has sufficient justification for saying objects only exist in perception, which makes the rest irrelevant.

    You can’t appeal to the lack of justification for objects existing in perception because, according to Kant and you, we cannot know anything about the things-in-themselves: we can only know the phenomena. If there’s no justification for saying there are objects (of which you can’t make if you can’t claim stuff about things-in-themselves), then we simply cannot know. If we cannot know, then you can’t say there is object permanence. My point is that you cannot refute (even probabilistically) the claim that your mind produces the objects without making an assumption about the things-in-themselves, which you aren’t supposed to be able to do.

    Semantics, huh? Why don’t we just agree that if you know a thing, you’ve experienced it.

    I said:

    It can agree with this, as a matter of semantics, if you are saying that possible knowledge is that which one experiences; but then this just pushes the question back: why can’t we say that possible knowledge goes beyond our experiences?

    If by “if you know a thing, you’ve experienced it”, you just mean that you’ve experienced something, then, sure, that is true; but it is an uninformative tautology.

    The question up for debate here is whether you have justification for claiming there are things-in-themselves that are being represented in that experience—not that having an experience is having an experience.

    Why wouldn’t that be true? The truth of that doesn’t affect the premise that if a thing is known it must have been an experience, and doesn’t affect possible experience.

    You can’t claim that possible knowledge goes beyond our experiences (quite frankly: your experiences) because that is non-phenomena, which are, by definition, things-in-themselves; and you cannot know anything about those.

    Of course. The categories are nothing but theoretical constructs. It is merely a logically consistent speculation that understanding relates pure conceptions to cognition of things. Pretty hard to experience a theory, right?

    But this is exactly what people who venture into understanding the things-in-themselves do! They speculate about them and come to the most reasonable conclusion in coherence with experience. So why think we cannot do that?

    Now, for me, this is exactly backwards. I mean…what comes first, the appearance of a thing, or the representation of it? Our understanding of the world is dictated by our representational faculties.

    The appearance, then the representation. We come to know that is the case from the other way around.

    We extrapolate that the representations we see are just that: representations. And there are appearances that come first to those representations. Our understanding of the world is dictated by our representational faculties, but that doesn’t mean we can’t give cogent accounts of beyond that; which includes the claim that we are beings that exists in a transcendent world with representative faculties.

    Ehhhhh…..we just have different ideas of what entails metaphysics.

    That’s fine! As long as we understand what each other are saying.

    While it may be fine to say it is understood for something to be beyond the possibility of all experience, it remains the case that understanding is not authorized to say what that something is

    Then you can’t claim that you have a mind. You can’t claim that you have representative faculties. You can’t claim that your representative faculties use pure conceptions. These are all beyond the possibility of all experience.

    Understanding cannot inform what things are not conditioned by the categories,

    But you cannot equally know, by your previous claim quoted above this one, that there are categories: they are likewise beyond the possibility of all experience.

    Yours wants the content of a conception as metaphysical, which is an exposition of it; mine wants that there are conceptions, including their content, not thought spontaneously as in understanding in conjunction with a synthesis of relations, but given complete in themselves from a pure a priori source.

    Correct. Which, under yours, then, you must be, by my lights, an epistemic solipsist. You must not know if there is object permanence, etc. because all your reprsentations are purely a priori and cannot be derived farther back than that. It just seems to me like an incredibly unparsimonious account of reality.

    Bob
  • Mww
    4.6k
    It sounds like you are saying there are minds which are of a mental substanceBob Ross

    I only said what my mind is not. I’ve said before I don’t hold that minds are anything beyond an object of reason, which negates that I may be what’s referred to as a substance dualist.

    But, traditionally, a mind is a conscious intelligence—a thinking subject which has qualia.Bob Ross

    Oh. A new tradition, then. The old one didn’t need qualia for conscious intelligence. Got along rather well without them, actually. A pair of shoes with a pretty shine, is still just a pair of shoes.

    I agree that the body is not a thing-in-itself, but the mind (or something else) must be.Bob Ross

    Ok. Why must it be? For a mind, or something else which serves the same purpose, to be a thing-in-itself makes necessary it is first and foremost, a thing. Says so right there in the name.

    Even if the mind is not a ‘thing’ in the sense of being of a physical substance, it is a ‘thing-in-itself’ of a mental substance.Bob Ross

    This looks like a way to force acknowledgement for the existence of a mind. The thing-in-itself is a physical reality, so if the mind is a thing-in-itself in a mental reality, then that’s sufficient reason to justify its existence? Which still requires an exposition for mental substance such that mind can emerge from it. Are you using Descartes for that exposition? It’s in Principia Philosophiae 1, 51-53, 1644, if you want to see how yours and his compare.

    It just seems to me like an incredibly unparsimonious account of reality.Bob Ross

    I am not accounting for reality; I’m accounting, by means of a logical methodology, reality’s relation to me.

    You’re correct, in that I don’t know any of those things you listed, in the same manner as I know the things of my experience. But I know with apodeictic certainty the conditions under which the relations logic obtains, and from which my experiences follow, do not contradict Nature, which is all I need to know. ‘Course, I might someday trust that logic so far that it kills me, but it hasn’t yet, so I must be doing something right. Or at least not wrong enough to sustain permanent damage.

    You want me to go further in my accounting, but I don’t see any need for it.
  • Mww
    4.6k
    ”…..if you know a thing, you’ve experienced it.”
    -Mww

    I said: (…) if you are saying that possible knowledge is that which one experiences….
    If by “if you know a thing, you’ve experienced it”, you just mean that you’ve experienced something…..

    The question up for debate here is whether you have justification for claiming there are things-in-themselves that are being represented in that experience—not that having an experience is having an experience.
    Bob Ross

    Do you see that neither of your follow-up’s relate to what I said?
    ……Possible knowledge, knowledge not in residence, cannot be from experience that is.
    ……To experience is not necessarily to know, but to know is necessarily to experience.

    Justification for claiming things-in-themselves are being represented in experience, should never be a question up for debate, and if it does arise as such, it can only be from a different conception of it. To represent a thing-in-itself in its original iteration, is self-contradictory, insofar as the thing-in-itself is exactly what is NOT developed in the human intuitive faculty for representing sensible things.
    ———-

    Our understanding of the world is dictated by our representational faculties, but that doesn’t mean we can’t give cogent accounts of beyond that….Bob Ross

    Then why isn’t such cogent account given by the understanding that’s already dictated our understanding of the world? If it can, then it hasn’t dictated as much as merely proposed, and if it can’t then its dictation is all it is capable of doing, which releases anything beyond it from being an object of it, which in turn means there won’t be a further cogent account.

    But I get it. Reason can always influence understanding by enabling thoughts or chain of thinking beyond that which is dictated by the representational faculties. The old, “what if…..” scenario, which only reason can initiate, and in so doing requests that understanding bend its own rules. Which is fine, obviously, in that empirical science advances in no other way, except for sheer accident. Thing is, though, empirical science is checked by either Nature or experience, whereas pure mental exercise has no such check, but relies on self-correction in the form of logical juxtaposition to synthetic a priori principles, like..…. “no, son, you cannot enclose a space with two straight lines. Don’t even go there.”

    So it turns out, not only does reason ask understanding to bend its own rules, but justifies the request because it has already bent its own principles. If that happens, there are no checks and balances left at all, and there manifests an intellectual free-for-all where anything goes, an “…embarrassment to the dignity of proper philosophy….”, so those old-time actual professional philosophers would have us know.
  • Bob Ross
    1.2k


    Hello Mww,

    I only said what my mind is not. I’ve said before I don’t hold that minds are anything beyond an object of reason, which negates that I may be what’s referred to as a substance dualist.

    I see. Would you say that your mind does not exist in the things-in-themselves? If so, then what other possible options (to you) are their for where it “resides”?

    Ok. Why must it be? For a mind, or something else which serves the same purpose, to be a thing-in-itself makes necessary it is first and foremost, a thing. Says so right there in the name.

    It has to be a ‘thing” (either of a mental or physical substance) if it to be distinguishable from nothing: only things which do not exist are not of a substance. Are you saying that ‘mind’ is just an emergent property from something else (that is the thing-in-itself)? I am having a hard time pinning down what you are saying here. Bottom line, to me, the mind, or whatever it is emergent from, must be traced back to something which is a thing-in-itself. If it is not itself the thing-in-itself, then it is an illusion. If it neither an illusion nor a thing-in-itself, then it doesn’t exist.

    This looks like a way to force acknowledgement for the existence of a mind.

    It’s meant to force acknowledge that the mind is of something. Either it is the thing-in-itself, emergent of a thing-in-itself, or it simply doesn’t exist.

    The thing-in-itself is a physical reality

    How could you know that if things-in-themselves are purely negative conceptions?

    Which still requires an exposition for mental substance such that mind can emerge from it.

    One could claim that something is eternal and of a mental substance. It doesn’t necessarily have to be emergent from. I am just trying to pin down what you think a mind is, and so far is seems like just ‘reason’ and ‘the unknown’.

    Are you using Descartes for that exposition? It’s in Principia Philosophiae 1, 51-53, 1644, if you want to see how yours and his compare.

    Thank you for the reference, but, unfortunately I have not read that nor was I able to parse your citation to find it in a free PDF version of the book. Could you perhaps include in a the excerpt if you already know what you are referencing? Otherwise, no worries.

    I am not accounting for reality; I’m accounting, by means of a logical methodology, reality’s relation to me.

    yes, but you are fundamentally saying that reality, true reality, is beyond our epistemic limits. And this entails a long of, in my opinion, unparsimonous positions (e.g., cannto know of object permanence, minds, one’s mind being a representative faculty, etc.).

    But I know with apodeictic certainty the conditions under which the relations logic obtains, and from which my experiences follow, do not contradict Nature, which is all I need to know.

    How do you know it doesn’t contradict nature if you can’t know anything about true nature? These are the kinds of weird implications I see if I were to commit myself to transcendent idealism.

    Do you see that neither of your follow-up’s relate to what I said?

    No I don’t see that. But let me try to address:

    Possible knowledge, knowledge not in residence, cannot be from experience that is.

    Why can possible knowledge not be from experience? Wouldn’t you have to know that your mind isn’t producing the objects? And wouldn’t that requiring knowledge of the things-in-themselves?

    To experience is not necessarily to know, but to know is necessarily to experience.

    Agreed.
    Justification for claiming things-in-themselves are being represented in experience, should never be a question up for debate, and if it does arise as such, it can only be from a different conception of it.

    Why??? This is just a flat assertion: I am asking exactly that! If you can’t know anything about the things-in-themselves, if you are truly trapped within your phenomenal experience, then why would you even know there are things-in-themselves? It seems like you are just appealing to an intuition here. I have no problem with that BUT I can do the same exact thing about things-in-themselves.

    To represent a thing-in-itself in its original iteration, is self-contradictory, insofar as the thing-in-itself is exactly what is NOT developed in the human intuitive faculty for representing sensible things.

    Again: why??? It’s just flatly asserted that we can’t question (e.g., ‘it’s self-contradictory’) that there are things-in-themselves, but all of my knowledge that I am represented something is phenomena! I thought those shouldn’t tell us anything about the things-in-themselves? The schema Kant as come up with here undermines itself to me.

    Then why isn’t such cogent account given by the understanding that’s already dictated our understanding of the world?

    What do you mean? I didn’t follow this part.

    So it turns out, not only does reason ask understanding to bend its own rules, but justifies the request because it has already bent its own principles

    If I am understanding you correctly, then you are using the “understanding” vs. “reason” semantics from Kant (which is fine). If so, then I would say that (1) your ability to acquire the knowledge of the ‘understanding’ is just metaphysics (and is no different than what I am doing) and (2) I reject Kant’s formulation of it as merely an exposition of ‘reason’ as opposed to the ‘understanding’. Maybe if I was convinced that we really had these twelve categories of the understanding and such, then I would be metaphysically cut off from further inquiry beyond that.

    Maybe expound whatever proof you found convincing for Kant’s twelve categories: that might help me understand better.

    If that happens, there are no checks and balances left at all, and there manifests an intellectual free-for-all where anything goes, an “…embarrassment to the dignity of proper philosophy….”, so those old-time actual professional philosophers would have us know.

    As of now, I don’t buy this. We use parsimony, coherence, intuitions, reliability, consistency, empirical adequacy, etc. and this doesn’t require us to limit ourselves to transcendental investigations.

    Bob
  • Mww
    4.6k
    I have no problem with that BUT I can do the same exact thing about things-in-themselves.Bob Ross

    I don’t think so. Not exactly. You can do as you wish, re: appeal to intuition, but you must first treat the thing-in-itself differently, such that it is all and only that which appears to be represented in intuition as a phenomenon. The established condition is that it is not, therefore you must show that it is. In order to do that, you must treat it differently. Which is fine, you are certainly authorized by your reason a priori. Just, not in accordance with Kantian transcendental philosophy.

    Why can possible knowledge not be from experience?Bob Ross

    Experience is present or past; possible knowledge is future. Possible knowledge requires possible experience. Seems pretty cut and dried to me.

    We use parsimony, coherence, intuitions, reliability, consistency, empirical adequacy, etc. and this doesn’t require us to limit ourselves to transcendental investigations.Bob Ross

    Yeah, but I want to know if all those reduce to something that grounds them all, or if there is not. For that investigation, a transcendental method, insofar as a priori cognitions are the only way for my determinations with respect to those wishes to manifest and a transcendental method proves the validity of them, I am well served by it.

    If I am understanding you correctly, then you are using the “understanding” vs. “reason” semantics from Kant (which is fine). If so, then I would say that (1) your ability to acquire the knowledge of the ‘understanding’ is just metaphysics (and is no different than what I am doing) and (2) I reject Kant’s formulation of it as merely an exposition of ‘reason’ as opposed to the ‘understanding’Bob Ross

    (1)….correct. I don’t acquire the knowledge of understanding; it is methodologically given as a faculty contained in and used by a speculative system, and is thereby just metaphysics;
    (2)….reject to your own satisfaction. That doesn’t detract from the ground of the formulation which shows what the opposition is.
    ————-

    Maybe expound whatever proof you found convincing for Kant’s twelve categories: that might help me understand better.Bob Ross

    “…. Transcendental analytic is the dissection of the whole of our à priori knowledge into the elements of the pure cognition of the understanding. In order to effect our purpose, it is necessary: (1) That the conceptions be pure and not empirical; (2) That they belong not to intuition and sensibility, but to thought and understanding; (3) That they be elementary conceptions, and as such, quite different from deduced or compound conceptions; (4) That our table of these elementary conceptions be complete, and fill up the whole sphere of the pure understanding. Now this completeness of a science cannot be accepted with confidence on the guarantee of a mere estimate of its existence in an aggregate formed only by means of repeated experiments and attempts. The completeness which we require is possible only by means of an idea of the totality of the à priori cognition of the understanding, and through the thereby determined division of the conceptions which form the said whole; consequently, only by means of their connection in a system. Pure understanding distinguishes itself not merely from everything empirical, but also completely from all sensibility. It is a unity self-subsistent, self-sufficient, and not to be enlarged by any additions from without. Hence the sum of its cognition constitutes a system to be determined by and comprised under an idea; and the completeness and articulation of this system can at the same time serve as a test of the correctness and genuineness of all the parts of cognition that belong to it…..”

    Convinced of a proof grounded in an idea? Nahhhh….no more than persuaded, and that in conjunction with his claim that he’s thought of everything relevant, and needs nothing from me to complete the thesis. For me to think he could have done better, or that he trips all over himself, implies I’m smarter than he, which I readily admit as hardly being the case.

    Funny, though, innit? To help you understand? You realize, don’t you, that is beyond my abilities? No matter what anybody says in attempting to help you, you’re still on your own after they’ve said whatever it is they going to say. And because you’ve rejected some parts, it isn’t likely you’re going to understand the remainder as a systemic whole, which necessarily relates to the parts rejected.

    My interest here is waning , sorry to say.
  • Bob Ross
    1.2k


    Hello Mww,

    My interest here is waning , sorry to say.

    Absolutely no worries! We can stop at anytime that you deem fit.

    Convinced of a proof grounded in an idea? Nahhhh….no more than persuaded, and that in conjunction with his claim that he’s thought of everything relevant, and needs nothing from me to complete the thesis. For me to think he could have done better, or that he trips all over himself, implies I’m smarter than he, which I readily admit as hardly being the case.

    I didn’t see a proof in that quote of the 12 categories of the understanding but, rather, a summary of transcendental philosophy.

    Funny, though, innit? To help you understand? You realize, don’t you, that is beyond my abilities? No matter what anybody says in attempting to help you, you’re still on your own after they’ve said whatever it is they going to say. And because you’ve rejected some parts, it isn’t likely you’re going to understand the remainder as a systemic whole, which necessarily relates to the parts rejected.

    Come on Mwww! (; If you are convinced that there are twelve categories, then you should be able to articulate the proof that convinced you. I am not asking for a super meticulous exposition of all of transcendental philosophy: I already understand the basic transcendental idealistic context for the argument.

    I am just curious, within that context, why you would think there are these twelve categories. Why not, for example, hold there is one: the PSR of becoming. Why place the function and purpose of reason in the understanding, such as concepts?

    Bob
  • Janus
    15.6k
    If you are convinced that there are twelve categories, then you should be able to articulate the proof that convinced you.Bob Ross

    I can't help interjecting here, since I have already explained this to you. The categories of understanding are identifiable simply by reflecting on the ways we experience and judge things; nothing at all to do with the thing-in-itself.

    The problem is, Bob, I don't think you are listening to anyone else.
  • Mww
    4.6k
    I didn’t see a proof in that quote….Bob Ross

    Because, as you say, it’s a summary, or an abstract, sort of, hence there isn’t a proof per se. There is only, in the text that follows, an affirmative argument for something, at the time, that had never even been considered by any of his peers.

    quote="Bob Ross;817972"]……you should be able to articulate the proof that convinced you.[/quote]

    My conviction regarding the fact of the categories is irrelevant. I’m sufficiently persuaded by the affirmative argument to think he’s come up with a perfectly fascinating metaphysical theory. That’s it.

    “…. If a judgement is valid for every rational being, then its ground is objectively sufficient, and it is termed a conviction. If, on the other hand, it has its ground in the particular character of the subject, it is termed a persuasion.….”

    I could articulate the argument, but all I’d doing is reading the book to you. To be as fair as possible, re: not imbue my interpretive subjectivity into a text, you should study it for yourself. Cut out the middleman, so to speak.

    Now you may say you’ve already done that, but your interpretation is so different from mine that you’re just looking for clarity. But what if I’ve got it all wrong? Then you’re right where you started, anyway, left with your own understanding. As it should be.
    ————-

    Addendum: Fancy-talk for dammit!! I saw this but forgot to mention it:

    On your “Of The Originally Synthetical Unity of Apperception” quote, the very next line after what you posted, shoots your argument in the foot. The “I think” is a representation, therefore not a thing-in-itself, which is, of course, never that which can be represented.

    “….. But this representation, “I think,” is an act of spontaneity; that is to say, it cannot be regarded as belonging to mere sensibility. I call it pure apperception, in order to distinguish it from empirical; or primitive apperception, because it is self-consciousness which, whilst it gives birth to the representation “I think,” must necessarily be capable of accompanying all our representations…..”

    Just sayin’…..
bold
italic
underline
strike
code
quote
ulist
image
url
mention
reveal
youtube
tweet
Add a Comment